Which of the following values are in the range of the function graphed below?
Check all that apply.
O B. -1
O C. 5
D. 2
E. 1

Which Of The Following Values Are In The Range Of The Function Graphed Below?Check All That Apply.O B.

Answers

Answer 1

Answer:

I believe the answer should be 0 to -1

Step-by-step explanation:

the range of a function is the lowest y value to the highest y value of that function. so in that example, the lowest y value you have on that curved line is at 0, and the highest y value you have on that curved line is 1, so the range should be from 0 to 1, or {0, 1}.


Related Questions

Choose the smallest number 3 1/8 or 10/3

Answers

Answer:

10/3

Step-by-step explanation:

31/8= 3.8

10/3= 3.3

Hello, Brainly community!

This question is for all of those Calculus people out there.

The volume of a swimming pool is changing with respect to time, such that the volume is given by W(t), where W(t) is measured in cubic centimeters and t is measured in seconds. A tangent line is shown for W(t) at t = 3 seconds. Determine the best estimate for the value of the instantaneous rate of change of W(t) when t = 3.
(I've narrowed down the answer choices to 2, and just really need to find the right way of thinking to find the answer)

(A) W(lim t) as t goes to 3.
(B) [W(3.1) - W(2.9)] / 0.2.

Thank you in advance!

Answers

Answer:

(B)  [tex]\displaystyle \frac{W(3.1) - W(2.9)}{0.2}[/tex]

General Formulas and Concepts:

Calculus

Limits

Derivatives

The definition of a derivative is the slope of the tangent line.

Derivative Notation

Instantaneous Rates

Tangent Line: [tex]\displaystyle f'(x) = \frac{f(b) - f(a)}{b - a}[/tex]

Step-by-step explanation:

Since we are trying to find a rate at which W(t) changes, we must find the derivative at t = 3.

We are given 2 close answer choices that would have the same numerical answer but different meanings:

(A)  [tex]\displaystyle \lim_{t \to 3} W(t)[/tex](B)  [tex]\displaystyle \frac{W(3.1) - W(2.9)}{0.2}[/tex]

If we look at answer choice (A), we see that our units would simply just be volume. It would not have the units of a rate of change. Yes, it may be the closest numerically correct answer, but it does not tell us the rate at which the volume would be changing and it is not a derivative.

If we look at answer choice (B), we see that our units would be cm³/s, and that is most certainly a rate of change. Answer choice (B) is also a derivative at t = 3, and a derivative tells us what rate something is changing.

∴ Answer choice (B) will give us the best estimate for the value of the instantaneous rate of change of W(t) when t = 3.

Topic: AP Calculus AB/BC (Calculus I/I + II)

Unit: Differentiation

Book: College Calculus 10e

someone help me for this algebra task please

Answers

i just need to answer questions

find the domain of f(x)=sec(2x)

Answers

Answer:

*Refer the image attached

Step-by-step explanation:

*Refer the image attached

pls help me don't know what to do

Answers

Answer:

x=15

Step-by-step explanation:

The 60 degree angle and the (x+45) degree angle are both the same degree because they are vertical angles.

So to solve, just subtract 45 from 60

60-45=15

That's your answer!

Hope this helps!

Arturo is building a flower bed in the shape of a right triangle. The hypotenuse of the right triangle is 13 feet. One of the legs needs to be 7 feet longer than the other leg.

Which equation can be used to find x, the length of the shorter leg?

x(x + 7) = 13
x(x + 7) = 169
x2 + (x + 7)2 = 13
x2 + (x + 7)2 = 169

Answers

Answer:

x(x + 7) = 169

Step-by-step explanation:

We are given that

Let length of shorter leg of right triangle = x ft

Length of other leg = x+7 ft

Hypotenuse of right triangle = 13 ft

We have to find the equation which can be used to find x, the length of shorter leg.

By pythagorous theorem

(Hypotenuse)^2=(Base)^2+(Perpendicular\;side)^2

Substitute the values then, we get

(13)^2=x^2+(x+7)^2

x^2+(x+7)^2=169

This is required equation which can used to find x.

Answer:

d. x2 + (x + 7)2 = 169

Step-by-step explanation:

use Pythagorean theorem, substitute in the given values, and you get this

D=22/7×d-90 Solve the equation
Find D​
Fast!

Answers

Answer:

D=22-90+22d/7

Step-by-step explanation:

D=22/7×d-90

D=-90+22d/7

Answer is- d=-630/-22x+7

PLS HELP SOON WILL MARK BRAINLYEST

A railroad tunnel is shaped like a semi-ellipse, as shown below. A semiellipse is shown on the coordinate plane with vertices on the x axis and one point of intersection with the positive y axis. The height of the tunnel at the center is 35 ft, and the vertical clearance must be 21 ft at a point 8 ft from the center. Find an equation for the ellipse.

Answers

According to the question

b= 35 and (8,21) lies on the ellipse

After calculation we get a= 10

equation for the ellipse.

[tex] \frac{ {x}^{2} }{100} + \frac{ {y}^{2} }{1225} = 1[/tex]

According to the number line, what is the distance between points A and B?

0 6 units
7 units
O 12 units
O 14 units

Answers

Answer:

14 units

Step-by-step explanation:

A = - 2, B = 12

Therefore,

d(A, B) = 12 - (-2) = 12 + 2 = 14 units

If a bus travel for 120 minutes at a speed of 75 kilometers per hour how far has the bus traveled?

Answers

Answer:

150 km

Step-by-step explanation:

Put the minutes into hours 120min is 2 hours.

Distance = speed * time

Distance = 75 * 2

Distance = 150

Answer:

150 kilometers

Step-by-step explanation:

if the bus is going 75 kilometers an hour and they traveled for 120 minutes (exactly 2 hours) then you would just multiply 75 by 2 to get 150 kilometers total.

The expression 2x and x² have the same value for only two values of x. What are these values?​

Answers

Answer:

0 and 2

Step-by-step explanation:

Evaluate 2y when y = 6y

Answers

I believe the answer is 12y
im sure the answer is 12y

Solve for x

Help me please

Answers

Answer:

x = 42.5

Step-by-step explanation:

A triangle inscribed inside of a circle using the diameter of the circle as it's hypotenuse is a right triangle ( a 90 degree triangle ) where the angle opposite of the hypotenuse is the 90 degree angle ( angle C )

Hence angle C = 90

If angle C = 2x + 5

Then 2x + 5 = 90

Notice that we just created an equation that we can use to solve for x

We now solve for x algebraically

2x + 5 = 90

Step 1 subtract 5 from both sides

2x + 5 - 5 = 90 - 5

Outcome: 2x = 85

Step 2 divide both sides by 2

2x/2 = 85/2

Outcome: x = 42.5

A parabola opens upward. The parabola goes through the point (3,-1),
and the vertex is at (2,-2).

Find the value of A for the parabola. Show your work. Use Part 1 and 2 to write the equation of the parabola.

Answers

Answer:

a=1

Step-by-step explanation:

Hopefully this helps :)

The equation of the parabola is: y = (x - 2)² - 2. Finding the value of A

The vertex of the parabola is at (2,-2). Since the parabola opens upward, the equation of the parabola will be of the form:

y = A(x - 2)² - 2

We can plug the point (3,-1) into this equation to find the value of A.

-1 = A(3 - 2)² - 2

Simplifying the right side of the equation, we get:

-1 = A - 2

Adding 2 to both sides of the equation, we get:

1 = A

Therefore, the value of A is 1.

Writing the equation of the parabola

The equation of the parabola is:

y = (x - 2)² - 2

To know more about parabola:

https://brainly.com/question/11911877

#SPJ2

It took Sarah 4 days to write a paper, she wrote 12 pages on day 1, 15 pages on day 2 and 9 pages on day 3. If she wrote 12 pages per day , how many pages did she write on the fourth day ?

Answers

Answer:

Option A

Step-by-step explanation:

Sarah took 4 days to write a paper.

She wrote 12 pages per day, so total number of pages she wrote in 4 days = 12 × 4

= 48 pages

On day 1, she wrote number of pages = 12

On day 2, she wrote number of pages = 15

On day 3, she wrote number of pages = 9

On day 4, she wrote number of pages = P

She wrote total number pages in 4 days = 12 + 15 + 9 + P

                                                                    = 36 + P

Therefore, P + 36 = 48

P = 48 - 36

P = 12

She wrote 12 pages on day 4.

Option A is the answer.

An 80-mile trip is represented on a gridded map by a directed line segment from point M(3, 2) to point N(9,
13). What point represents 60 miles into the trip? Show your work and explain your reasoning.

Answers

Answer:

Step-by-step explanation:

If the distance from M to N is 80 miles and we want to find the coordinates of the point 60 miles into the trip, we are looking for the point 3/4 of the way from M to N, since 60 is 3/4 of 80. This is the process:

First we need to consider that MN is a directed vector. We first find the components of the directed vector, which is found in the change in x and the change in y. First step, then, looks like this:

<Δx, Δy> = <9-3, 13-2> = <6, 11> We will call those the x and y components of the vector (which comes from vector study in both physics and math, so if you don't understand that, it's ok! Just follow the process here and you'll be fine). Knowing that 60 is 3/4 of the way from M to N, we find 3/4 of both the x and y components, like this:

[tex]<\frac{3}{4}(6), \frac{3}{4}(11)>[/tex] which will give us 3/4 of the change in x and 3/4 of the change in y:

[tex]<\frac{18}{4},\frac{33}{4}>=<4.5,8.25>[/tex]

Since we are going 3/4 of the way from M to N, we add that component to the x and y coordinates of M, giving us 3/4 of the way from M to N, which translates to 60 miles of the 80 mile trip:

(3 + 4.5, 2 + 8.25) = (7.5, 10.25)

Those are the coordinates of the point that represents 60 miles into the 80 mile trip.

The center of the circle is at the point
, and its radius is
units. The equation of this circle in standard form is
.

Answers

Is there a picture that goes with this?

Find ∠MPN

Help me please

Answers

Answer:

[tex]22^{\circ}[/tex]

Step-by-step explanation:

Line [tex]\overline{PM}[/tex] is a diameter of the circle because it passes through the circle's center O. Therefore, arc [tex]\widehat{PLM}[/tex] must be 180 degrees, as these are 360 degree in a circle.

We can then find the measure of arc [tex]\widehat{LM}[/tex]:

[tex]\widehat{LP}+\widehat{LM}=180^{\circ},\\92^{\circ}+\widehat{LM}=180^{\circ},\\\widehat{LM}=88^{\circ}[/tex]

Arc [tex]\widehat{LM}[/tex] is formed by angle [tex]\angle LPM[/tex]. Define an inscribed angle by an angle with a point on the circle creating an arc on the circumference of the circle. The measure of an inscribed angle is exactly half of the measure of the arc it forms.

Therefore, the measure of [tex]\angle LPM[/tex] must be:

[tex]m\angle LPM=\frac{88}{2}=44^{\circ}[/tex]

Similarly, the measure of [tex]\angle LNP[/tex] must be:

[tex]m\angle LNP=\frac{92}{2}=46^{\circ}[/tex]

Angles [tex]\angle LPM[/tex] and [tex]\angle MPN[/tex] form angle [tex]\angle LPN[/tex], which is one of the three angles in [tex]\triangle LPN[/tex]. Since the sum of the interior angles of a triangle add up to 180 degrees, we have:

[tex](\angle MPN+\angle LPM)+\angl+ PLN+\angle LNP=180^{\circ},\\\angle MPN+44+46+68=180,\\\angle MPN=180-44-46-68,\\\angle MPN=\boxed{22^{\circ}}[/tex]

Which operation will solve the following word problem? Jeff earns $14.00 per hour, Tom earns half as much as Jeff. How much does Tom earn per hour?


Multiplication


Subtraction


Addition


Division

Answers

Answer:

The correct option is (d).

Step-by-step explanation:

Given that,

Jeff earns $14.00 per hour.

Tom earns half as much as Jeff.

We need to find the amount earn by Tom per hour.

Tom's amount = Jeff's amount/2

So,

[tex]T=\dfrac{14}{2}\\\\T=\$7[/tex]

So, Tom earn $7 per hour. Hence, division operation is used. Jeff's amount is divided by 2.

A certain jet can reach an altitude of 8000 ft while flying 16000 ft through the air. what is the angle of the elevation that the plane is flying? Round to the nearest degree.​

Answers

Answer:

27 degrees

Step-by-step explanation:

First, we can draw this out. Since the altitude is 8000, we can make that the height, and it goes 16000 feet through the air, that can be the length. I have drawn this out, as shown in the image. Note that when drawing, because the airplane is going up and forward, that path should represent the hypotenuse. The angle of elevation is the angle connecting the length and the path, as shown by the angle x in the picture.

We know than tan(x) = opposite/adjacent, so tan(x) here = 8000/16000 = 1/2. Then, arctan(1/2) =x = 27 degrees

lidentify the domain of the function shown in the graph
O A 15257
O B. 19334
O C. 221
O D. All real numbers

Answers

Answer:

B.

Step-by-step explanation:

the visible line is the defined function.

this line goes from x=1 to x=4, and has the functional results from y=1 to y=7.

the domain is the valid interval of the input variable (typically x), while the range is the valid inescapable of the result variable (typically y).

so, B is the right answer.

What is the quotient of (x^3 - 3x^2 + 3x - 2) ÷ (x^2 - x + 1)?
O x - 2
O x + 2
O x- 4
O x + 1

Answers

Answer:

x-2

The choose (1)

Step-by-step explanation:

(x³-3x²+3x-2)÷(x²-x+1)

(x-2)(x²-x+1) ÷ (x²-x+1)

Delete (x²-x+1)

so = (x-2)

2 1/4 x 3 1/5 brainliest

Answers

Answer:

36/5

Step-by-step explanation:

9/4×16/5

144/20

36/5

hope this is helpful

Answer:

[tex]7\frac{1}{5}[/tex]

Step-by-step explanation:

1. start by turning the fractions improper fractions:

[tex]2\frac{1}{4} =\frac{9}{4}[/tex]

[tex]3\frac{1}{5} =\frac{16}{5}[/tex]

2. then multiply them together:

[tex]\frac{9}{4}[/tex] x [tex]\frac{16}{5}[/tex] = [tex]\frac{144}{20}[/tex]

3. then simplify the fraction:

[tex]\frac{144}{20}[/tex][tex]=\frac{36}{5}[/tex]

4. turn it into a proper fraction:

[tex]\frac{36}{5} =7\frac{1}{5}[/tex]

There are 35 times as many students at Wow University as teachers. When all the students and
teachers are seated in the 8544 seat auditorium, 12 seats are empty. How many students attend
Wow University?

Answers

Given:

There are 35 times as many students at Wow University as teachers.

When all the students and teachers are seated in the 8544 seat auditorium, 12 seats are empty.

To find:

The total number of students.

Solution:

Let x be the number of teachers at Wow University. So, the number of student is :

[tex]35\times x=35x[/tex]

When all the students and teachers are seated in the 8544 seat auditorium, 12 seats are empty.

[tex]x+35x=8544-12[/tex]

[tex]36x=8532[/tex]

[tex]x=\dfrac{8532}{36}[/tex]

[tex]x=237[/tex]

The number of total students is:

[tex]35x=35(237)[/tex]

[tex]35x=8295[/tex]

Therefore, the total number of students is 8295.

Cayden has several screws on a scale, and the scale reads 80.955 cayden add 1 more screw and the scale reads 84.81

Answers

Answer:

-3.855

Step-by-step explanation:

Not sure if you need the weight of each screw! but you just subtract 80.955-84.81

Answer:

1

Step-by-step explanation:

itnisbbsndnfnfnfnnfnfnfncjcjccjcj

What’s the equation of the line that passes through the point (-4,4) and has a slope of 3/4

Answers

Answer:

y-y1=m(x-x1)

y-4=3/4(x+4)

y=3/4x+7

A cinema is doing a promotion to celebrate their 50th anniversary for 1 week. They give

away a free drink to every 98th customer, a free bag of popcorn to every 112th customer and

a free cinema ticket to every 224th customer. Which lucky customer will be the first to

receive all 3 items?​

Answers

Answer:

1,568 customer

Step-by-step explanation:

Find the lowest common multiple of 98, 112, and 224

98 = 98, 196, 294, 392, 490, 588, 686, 784, 882, 980, 1078, 1176, 1274, 1372, 1470, 1568, 1666

112 = 112, 224, 336, 448, 560, 672, 784, 896, 1008, 1120, 1232, 1344, 1456, 1568, 1680, 1792, 1904

224 = 224, 448, 672, 896, 1120, 1344, 1568, 1792, 2016, 2240

The lowest common multiple of 98, 112, and 224 is 1568

Therefore, the 1,568th customer will be the first to receive all 3 iitem

A statistics professor asked students in a class their ages. On the basis of this information, the professor states that the average age of all the students in the university is 24 years. This is an example of

Answers

Answer:

propbability ???

Step-bp explanation:

Answer:

Step-by-step explanation:

This is an example of a statistical mean.

Find value of x.
A. 110
B. 47
C. 68
D. 112

Answers

Answer:

B

Step-by-step explanation:

The sum of the inner angles of a quadrilateral is 360 degrees

135 + 110 + 68 + x = 360

313 + x = 360

x = 47 degrees

Answer:

47

Step-by-step explanation:

whole thing is 360 degrees

68 + 110 + 135 = 313

360 - 313 = 47

x looks small too (if you had to guess in a multiple choice question)

The area of the rectangular sandbox at Dave's school is 117 square feet. The sandbox has a width of 9 feet as shown in the diagram. What is the perimeter of the sandbox?

Answers

Answer:

ay bru ima tell yu dhis rn is c

Step-by-step explanation:

Other Questions
what is the mid point of AB? A man cannot be a carrier for hemophilia. Why? Problem is in the picture below a body and spirit, he repeated, the body lady is like ahouse The question, "What are the distinguishing characteristics of effective leaders?" sparked which approach to the study of leadership? An elected government official is interested in the opinion of teachers in her voting area. She randomly selected five schools at random from the 20 schools in her area and then interviews each of the teachers in those five schools. The government official is using Write a function that takes six arguments of floating type (four input arguments and two output arguments). This function will calculate sum and average of the input arguments and storethem in output arguments respectively. Take input arguments from the user in the main function. Which of the following proteins could be used to carry proteins across thecell membrane? There are twelve shirts in my closet. Five are red, four are blue, and three are green. What isthe probability that I choose a red or blue shirt to wear tomorrow?O 65%0 75%0 80%60%58% 40 pointsanad brainliest for this, it's really important! Helpppp me plzzzzzzxzz HELP ASAP 10 POINTS AND BRAINLIST Someone please answer this which best describes a difference between transcription and dna replication Correct the sentence: The translation of subsidiary company as cng ty con used the Modulation technique from a part to the whole. Solids diffuse because the particles cannot move.A. CanB. Not enough infoC. CannotD. Sometimes will What was the original price of the car? Show all work ssonssts & QuizzesessagesWhat mistake(s) can you identify in this graph?1. Poor scale, wasted spaceII. No units for independent variableIII. No units for dependent variableIV. One axis uses decimals and the other does not.140astant TeacherSradebook12010080volume of gas produced in 1 minute604020o0.03.00.5 1.0 1.5 2.0 2.5acid concentration (M)Type here to searchoHE Which of the following is true about an opportunity cost? Multiple Choice they are recorded in the accounting records of the company. is a cost that has already been incurred and can not be changed by any decision. is the potential benefit given up when one alternative is selected over another. should never be considered in decision-making. Which statement best describes how pigments affect what colors humanssee?